Potential difference across the ends of a solenoid

In summary: You are right. The first integration is to find the area of the cyclic section and the second is to find the angle of the flux. But the second integration is "neutralized" when you take the time derivative of the flux.
  • #1
songoku
2,294
325
Homework Statement
A disc of area ##A## rotates at angular frequency ##\omega## at the center of long solenoid of length L and having N turns, carrying current I. The plane of the disc is normal to magnetic flux. The rotation rate is adjusted so that the emf generated between the center of the disc and its rim is 10% of the potential difference across the ends of the solenoid. Which expression gives the potential difference across the ends of the solenoid.
a) ##10 \mu_0 NIA \omega##
b) ##1.6 \mu_0 NIA \omega / L##
c) ##0.1 \mu_0 NIA \omega##
d) ##0.016 \mu_0 NIA \omega /L##
Relevant Equations
B at center of solenoid = ##\mu_0 NI / L##

##E = - \frac{d \phi}{dt }##
Emf between the center of the disc and its rim
= ##- \frac{d \phi}{dt }##
= ##-B.A \frac{d \cos (\omega t)}{dt}##
=##\mu_0 \frac{N}{L} I A \omega \sin (\omega t)##

Potential difference across the ends of the solenoid = ##10\mu_0 \frac{N}{L} I A \omega \sin (\omega t)##

Is this correct? How to continue until I get the answer on the options?

Thanks
 
  • Like
Likes Delta2
Physics news on Phys.org
  • #2
I don't understand how you get the trig terms. There is nothing in the question that varies with time or angle.
Hint: consider a small radial element of the disc of length dr. What is the p.d. along it?
 
Last edited:
  • Like
Likes songoku and Delta2
  • #3
Read about Faraday Homopolar generator to find out how to calculate EMF of the disc.
 
  • Like
Likes songoku
  • #4
haruspex said:
I don't understand how you get the trig terms. There is nothing in the question that varies with time or angle.
I use: ##\phi = BA \cos \theta## and I imagine ##\theta## will change as the disc rotates. Or maybe from the question " The plane of the disc is normal to magnetic flux" means that ##\theta## is constant? If ##\theta## is constant, my imagination about the direction rotation is wrong.

I imagine it like this: the plane of the disc is vertical and it rotates with vertical axis of rotation through the center of the disc. Or maybe the rotation is just like the rotation of bicycle wheel (with the plane always perpendicular to magnetic field produced by solenoid)?

What change causes the change in magnetic flux (to produced induced emf)? I thought the only possible change is ##\theta## since the value of B and A are constant. If the rotation of the disc is just like bicycle wheel, wouldn't the magnetic flux on the disc be constant, hence no emf?

Hint: consider a small radial element of the disc of length dr. What is the p.d. along it?
I need to use integration to solve this problem? I am not sure what is the p.d. along a small radial element of the disc since I have no idea what causes the emf to be produced.

Thanks
 
  • #5
Delta2 said:
Read about Faraday Homopolar generator to find out how to calculate EMF of the disc.
I will google it
 
  • #6
songoku said:
the plane always perpendicular to magnetic field produced by solenoid)?
Yes.
songoku said:
the magnetic flux on the disc be constant, hence no emf?
See the answer at https://physics.stackexchange.com/questions/466143/emf-induced-in-a-rotating-disc, where it points out the difference between electric Lorentz force and magnetic Lorentz force.
songoku said:
I am not sure what is the p.d. along a small radial element of the disc since I have no idea what causes the emf to be produced.
Just think of it as a wire moving across the field. If it is at radius r from the axis of rotation and pointing away from the axis, how fast is it moving across the field?
 
  • Like
Likes songoku
  • #7
songoku said:
If the rotation of the disc is just like bicycle wheel, wouldn't the magnetic flux on the disc be constant, hence no emf
Yes the rotation is like a bicycle wheel with the magnetic field perpendicular to the plane of the wheel. Seemingly it violates Faraday's law of induction.

To apply Faraday's law correctly, consider a radius of the "bicycle wheel" and calculate how much surface area S it spans in time t as this radius rotates with constant angular velocity ##\omega##. Then calculate ##\frac{dS}{dt}## and the emf will be $$\mathcal{E}=\frac{d\Phi}{dt}=\mathbf{B}\frac{dS}{dt}$$.

The other approach is that of @haruspex where you go by Lorentz force law and integration of infinitesimal EMFs ##Bvdr=B\omega rdr##.
 
Last edited:
  • Like
Likes songoku
  • #8
songoku said:
I need to use integration to solve this problem?
As @Delta2 notes, thinking in terms of area swept out by a complete radius does the integration for you.
 
  • Like
Likes songoku
  • #9
haruspex said:
As @Delta2 notes, thinking in terms of area swept out by a complete radius does the integration for you.
My approach involves two integrations actually.

  • One with respect to the spatial variables to find the area of a cyclic section of angle ##\phi## and radius ##R##, which of course is already well known and the result is ##\frac{\phi}{2}R^2##, and
  • One integration with respect to the time variable to find the angle ##\phi##. But the latter integration is going to be "neutralized" when we take the time derivative of the flux, so no actual integrations needed at all.
 
  • Like
Likes songoku
  • #10
haruspex said:
Just think of it as a wire moving across the field. If it is at radius r from the axis of rotation and pointing away from the axis, how fast is it moving across the field?
speed = ##\omega## . r ?

Delta2 said:
My approach involves two integrations actually.
  • One with respect to the spatial variables to find the area of a cyclic section of angle ##\phi## and radius ##R##, which of course is already well known and the result is ##\frac{\phi}{2}R^2##, and
  • One integration with respect to the time variable to find the angle ##\phi##. But the latter integration is going to be "neutralized" when we take the time derivative of the flux, so no actual integrations needed at all.
I think I understand integration of first part but I don't understand the second part. I thought there is only one integration with respect to ##\phi## to find the area swept by "radius of bicycle wheel" and after that we take the time derivative to obtain emf induced.

Thanks
 
  • #11
The integration of the second part is more easy than you think, it is just that $$\phi=\int \omega dt$$ because by the definition of ##\omega## it is $$\omega=\frac{d\phi}{dt}$$ , what do you think?
In this example ##\omega## is constant in time so things are even more simple.
 
  • #12
Delta2 said:
The integration of the second part is more easy than you think, it is just that $$\phi=\int \omega dt$$ because by the definition of ##\omega## it is $$\omega=\frac{d\phi}{dt}$$ , what do you think?
In this example ##\omega## is constant in time so things are even more simple.
What I had in mind is like this:

$$\phi = \int B.dA = B \int dA = \frac 1 2 Br^2 \phi$$

Then:

$$E=-\frac{d\phi}{dt}=-\frac{d (\frac 1 2 Br^2 \phi)}{dt}=-\frac 1 2 Br^2 \omega$$

How to apply the second integration in my working? Thanks
 
  • Like
Likes Delta2
  • #13
You are right, no need to do second integration e hehe.

Btw to correct a small typo in your work, you use the same letter ##\phi## for the flux and the angle. You should use capital ##\Phi## for the flux.
 
  • Like
Likes songoku
  • #14
Thank you very much haruspex and Delta2. Happy new year
 
  • Like
Likes etotheipi and Delta2
  • #15
Happy New Year to you too!
 
  • Like
Likes songoku and etotheipi

1. What is potential difference?

Potential difference, also known as voltage, is the difference in electric potential energy between two points in an electric circuit.

2. What is a solenoid?

A solenoid is a coil of wire that carries an electric current and produces a magnetic field.

3. How is potential difference measured across the ends of a solenoid?

Potential difference can be measured using a voltmeter, which is connected across the ends of the solenoid to measure the voltage drop.

4. Why is there a potential difference across the ends of a solenoid?

The potential difference across the ends of a solenoid is due to the presence of a magnetic field created by the flow of electric current through the coil.

5. How does the potential difference across the ends of a solenoid affect its magnetic field?

The strength of the magnetic field produced by a solenoid is directly proportional to the potential difference across its ends. This means that increasing the potential difference will result in a stronger magnetic field.

Similar threads

  • Introductory Physics Homework Help
Replies
3
Views
155
  • Introductory Physics Homework Help
Replies
7
Views
82
  • Introductory Physics Homework Help
Replies
3
Views
172
  • Introductory Physics Homework Help
Replies
10
Views
271
  • Introductory Physics Homework Help
Replies
25
Views
1K
  • Introductory Physics Homework Help
Replies
1
Views
714
  • Introductory Physics Homework Help
Replies
14
Views
521
  • Introductory Physics Homework Help
Replies
2
Views
635
  • Introductory Physics Homework Help
Replies
6
Views
2K
  • Introductory Physics Homework Help
Replies
19
Views
808
Back
Top